The First National Bank recently acquired three mortgages from the failed Fargo Bank. It estimates that the chances of these mortgages being repaid are 0.6, 0.4, 0.25. Assuming independence, fine the probability that:_________.

Answers

Answer 1

Answer:

a. 0.06

b. 0.18

Explanation:

a) all of them will be repaid

b) none of them will be repaid

P1 = 0.6, P2= 0.4, P3 = 0.25

a. The Probability that all will be repaid = (P1∩P2∩P3)

= P1 * P2 * P3 (Since independent)

= 0.6*0.4*0.25

= 0.06

Thus, the Probability that all will be repaid is 0.06

b. The Probability that none of them will be repaid  = (1-P1)*(1-P2)*(1-P3)

= (1-0.6)(1-0.4)(1-0.75)

= 0.4 * 0.6 * 0.75

= 0.18

Thus, the Probability that none of them will be repaid is 0.18


Related Questions

Shawn goes to the library to pick up a copy of a common tax form used file taxes. Which of the following is MOST likely the form that Shawn got?

1099

W-2

1776

1040

Answers

Answer:

Shawn would choose form 1040 filing tax form.

Explanation:

The answer is D.) 1040

I got a picture of the answer.

Hope this helps!

Diversification is good for shareholders. So why shouldn't managers acquire firms in different industries to diversify a company?

Answers

Answer:

The definition would be defined in the clarification portion below, according to the particular context.

Explanation:

Even before managers accomplish diversification besides trying to create a conglomerate whilst also buying other corporations, it is almost always accomplished at a premium surrounded by white market rates because once shareholders could effectively achieve consolidation according to their own besides investing money throughout multiple organizations. Although it may be more difficult to accurately determine productivity in a conglomerate, authority costs will be lower as well as assets might well be apportioned around through segments incompetently.

You expect KT Industries (KTI) will have earnings per share of $3 this year and expect that they will pay out $1.50 of these earnings to shareholders in the form of a dividend. KTI's return on new investments is 15% and their equity cost of capital is 12% The expected growth rate for KTI's dividends is:_________

a. $12.50
b. ​$39.25
c. ​$33.35
d. ​$20.00

Answers

Answer:

Growth rate  = 7.50%

Explanation:

Given:

Return on investment = 15%

Retention ratio = [1.5 / 3] 100 = 50%

Find:

Growth rate

Computation:

Growth rate  = Return on investment*Retention ratio

Growth rate  = 15% x 50%

Growth rate  = 7.50%

Equipment costing $17,500 with an estimated salvage value of $1,180 and an estimated life of 4 years was purchased on October 31, 2019. Using the straight-line depreciation method, what is the amount of depreciation expense to be recorded at December 31, 2019?

Answers

Answer:

the depreciation expense recorded is $680

Explanation:

The computation of the depreciation expense under the straight-line method is shown below:

= (Purchase cost - residual value) ÷ (estimated life)

= ($17,500 - $1,180) ÷ ( 4 years)

= $4,080

Now the 2 months depreciation is i.e. from November to December

= $4,080 × 2 months ÷ 12 months

= $680

Hence, the depreciation expense recorded is $680

If a 7% increase in the price of cheese causes a 7% reduction in the total revenue received by cheese farmers, the demand for cheese is:A.Inelastic.B.Elastic.C.Unit elastic.D.Infinite

Answers

Answer:

C.Unit elastic

Explanation:

Unit elastic demand is the term that describes a scenario where a change in price causes a proportionate change in demand. It is one of the types of elastic demand. A good or service is said to have elastic demand if a small change in price causes a considerable change in the quantity demanded.

In the unit elastic demand, if a product price changes by a certain percentage, the demand will change by an equal percentage. In this scenario, a 7 percent price increase results in a 7 percent decrease in demand.

You are saving money for a down payment on a new house. You intend to place $7,500 at the end of each year for three years into an account earning 5% per year. At the end of the fourth year, you will place $10,000 into this account. How much money will be in the account at the end of the fourth year?

Answers

Answer:

$37,848.9

Explanation:

We can use the interest rate formula to figure out how much is in the account after the first 3 years. The interest rate formula is show below:

[tex]A = P (1 + r)^t[/tex]

Let me delineate what each part of this equation means:

A = The total amount

P = The initial amount of money put into the account

R = the interest rate

T = Time

The equation gives us the following:

You place $7,500 each year for three years The interest rate is 5%At the end of the 4th year $10,000 will be placed in the account

First, let's calculate the P in the equation.

You put $7,500 each year for 3 years, so multiply 7,500 by 3.

[tex](7,500) * (3) = 22,500[/tex]

Next, let's start putting everything into the equation, like this:

[tex]A = 22,500(1 + .05)^3[/tex]

(When doing interest rate you have to move the decimal over twice)

Now that we have the equation, let's solve it!

[tex]A = 22,500(1.05)^3\\A = 22,500(1.15763)\\A = 26,046.6[/tex]

After 3 years $26,046.6 is in the account.

But, don't forget the last part of the question!

But you have a fourth year too!

Add the $10,000 onto the $26,046.6

That equals $36,046.6.

Lets plug this back into the equation for the final year

[tex]A = 36046.6(1.05)^1\\A = 37848.9[/tex]

Thus, the final answer will be $37,848.9

Hope this helps!

- Kay :)

After saving the money for the four years and by adding $10,000 in the end of fourth year the money the amount that will be saved is $48,942.23.

What is Future Value?

The temporal value of money is based on the simple notion that one dollar today is worth more than one dollar in the future. This is because one can invest the dollar they have today and have it increase at a rate of return, or interest, over time.

The formula for future value is-

[tex]\begin{aligned}\text{FV}&=\text{CF}\times\dfrac{(1+r)^n-1}{\text{r}}+\text{FV}\\&=\$7,500\times\dfrac{(1+0.05)^4-1}{0.05}+\$7,500\\&=\$48,942.23\end{aligned}[/tex]

Thus, the future value by the end of the fourth year is $48,942.23.  

For further details about the future value refer to this link:

https://brainly.in/question/40202543

A political leader suggesting that an economic downturn will be cushioned by nondiscretionary fiscal policy is referring to:______. A) Tax policy and spending policy B) A progressive income tax and a welfare state C) Interest rates and the money supply D) Interest rates and tax rates

Answers

Answer:

Option A is the correct approach.

Explanation:

This is indeed a connection to compulsory tax and government expense stabilizers which weren't at the discretion including its government. Throughout the event of a recession, expenses are cut, rising discretionary income to something like the extent that the economic depression is pacified. Unemployment insurance, as well as other social programs, are consequently expanded without the clear intervention of the government

The other options offered are also not relevant to the scenario presented. So, the solution above is the right one.

Mr. Hopper expects to retire in 30 years, and he wishes to accumulate $1,000,000 in his retirement fund by that time. If the interest rate is 12 percent per year, how much should Mr. Hopper put into his retirement fund at the end of each year in order to achieve this goal? a) $8,287.32 b) $12,483.17 c) $4,143.66 d) $4,000.00

Answers

Answer:

Annual payment = $4,143.66 (Approx)

Explanation:

Given:

P = $1,000,000

r = 12% = 0.12

n = 30

Find:

Annual payment

Computation:

[tex]Annual\ payment=P[\frac{(1+r)^n-1}{r} ] \\\\Annual\ payment=1,000,000[\frac{(1+0.12)^{30}-1}{0.12} ] \\\\ Annual\ payment=4143.66[/tex]

Annual payment = $4,143.66 (Approx)

The act of quitting one's job or occupation, as well as one's active working life, is known as retirement.

Quasi can also be achieved by lowering work hours or workload. Many people decide to retire when they are too elderly or being unable to function due to medical issues.

Mr. Hopper should put the payment of $4,143.66 (Approx) into his retirement fund at the end of each year in order to achieve the goal.  

The provided information is:

P = $1,000,000

r = 12% = 0.12

n = 30

The calculation of the annual payment is shown below:

[tex]\text{Annual payment}= P\frac{(1+r)^{n}-1 }{r}[/tex]

[tex]\text{Annual payment}= 1000000[\frac{(1+0.12)^{30}-1 }{0.12}][/tex]

Annual payment = $4143.66

Therefore the correct option is C.

To know more about the calculation of the annual payment, refer to the link below:

https://brainly.com/question/24108530

A company has three product lines, one of which reflects the following results:Sales 215,000Variable expenses 125,000Contribution margin 90,000Fixed expenses 140,000Net loss (50,000)If this product line is eliminated, 60% of the fixed expenses can be eliminated and the other 40% will be allocated to other product lines. If management decides to eliminate this product line, the company's net income will:__________. a. increase by $50,000 b. decrease by $90,000 c. decrease by $6,000 d. increase by $6,000

Answers

Answer:

c. decrease by $6,000

Explanation:

The computation of the company net income is shown below:

Sales $215,000

Less: Variable expenses ($125,000)

Contribution margin $90,000

Less: Fixed expenses(traceable) ($84,000)   ($140,000 × 0.60)

Net Income $ 6,000

And, the given net loss is $50,000

The fixed expense is $56,000 ($140,000 × 0.40)

So, the net income would decrease by $6,000

Hence, the correct option is c.

Suppose the real gross domestic product (GDP) grows by 2% and inflation is equal to 3%, but there is no change in the velocity of money. based on the equation of exchange, by how much does the quantity of money change?

Answers

Answer:

the quantity of money growed to be 5%

Explanation:

The computation of the expected change in the quantity of money is shown below:

As we know that

Growth rate of money supply = Real GDP growth rate + inflation rate

= 2% + 3%

= 5%

Keeping the velocity be constant

Hence, the quantity of money growed to be 5%

We simply applied the above formula so that the correct value could come

And, the same is to be considered  

Flounder Inc. purchased land, building, and equipment from Laguna Corporation for a cash payment of $434,700. The estimated fair values of the assets are land $82,800, building $303,600, and equipment $110,400. At what amounts should each of the three assets be recorded?

Answers

Answer:

ok

I thinks it's ok because it's ok you get me

Expando, Inc., is considering the possibility of building an additional factory that would produce a new addition to their product line. The company is currently considering two options. The first is a small facility that it could build at a cost of $9 million. If demand for new products is low, the company expects to receive $9 million in discounted revenues (present value of future revenues) with the small facility. On the other hand, if demand is high, it expects $12 million in discounted revenues using the small facility. The second option is to build a large factory at a cost of $10 million. Were demand to be low, the company would expect $12 million in discounted revenues with the large plant. If demand is high, the company estimates that the discounted revenues would be $15 million. In either case, the probability of demand being high is 0.40, and the probability of it being low is 0.60. Not constructing a new factory would result in no additional revenue being generated because the current factories cannot produce these new products.
a. Calculate the NPV for the following: (Leave no cells blank - be certain to enter "0" wherever required. Enter your answers in millions rounded to 1 decimal place.)
Plans NPV
Small facility $ million
Do nothing million
Large facility million
b. The best decision to help Expando is:_______.
a. to build the large facility.
b. to build the small facility.
c. to do nothing.

Answers

Answer:

a)

small facility:

initial outlay = -$9,000,000

present value of expected cash flows = (0.6 x $9,000,000) + (0.4 x $12,000,000) = $10,200,000

NPV = $10,200,000 - $9,000,000 = $1,200,000

large facility:

initial outlay = -$10,000,000

present value of expected cash flows = (0.6 x $12,000,000) + (0.4 x $15,000,000) = $13,200,000

NPV = $13,200,000 - $10,000,000 = $3,200,000

b) the best option is:

a. to build the large facility.

the NPV of the large facility is significantly higher than the NPV of the smaller facility, while the required investment is not that different.

Some club members want to increase membership dues by $7.00.Other club members want to increase them by $3.00.They have reached an impasse on the issue,so they decide to split the difference and raise the dues by $5.00.What type of conflict style did the group use?
A) Accommodating
B) Collaborating
C) Competing
D) Avoiding
E) Compromising

Answers

Answer:

Option E (Compromising) would be the correct choice.

Explanation:

A conflict mediation method of consensus attempts to discover a reason to reasonably pleasing parties and from both sides of the debate. When it becomes more necessary to optimize a compromise than for the conclusion to always be perfect, a deadline is fast approaching, even at such an ongoing crisis, because you need a workable measure only for the moment, such style might be suitable to be using.

The other choices aren't relevant to the situation presented. Because otherwise, that is the right answer.

Which of the following did Judge Parker NOT mention as one of the principles of ethics?

Answers

Answer:

Remain open-minded

Explanation:

The term learning organization is used to describe organizations in the educational
and nonprofit industries.
True or false

Answers

Answer: False

Explanation: The term "Learning organizations" can be defined as a place where people continue learning and expand their capacity to see the results they desire.  According to Peter Sange, the one who coined the concept with his colleagues, Learning organization responds as a company that helps its employees to learn more.

An advertising agency wishes to estimate, with 99% confidence and to within $45, the mean amount small business owners would be willing to pay for a professionally produced web page for their businesses. Assuming a standard deviation of such amounts to be $250, the estimated minimum sample size needed to meet these criteria is about:__________

Answers

Answer:

b) 205

Explanation:

Option are "(a) 103 b) 205 c) 644 d) 97"

The estimated minimum sample size needed to meet these criteria is calculated below: n = (Z*σ/E)^2

n = (2.58 * 250 / 45)^2

n = (645 / 45)^2

n = (14.33)^2

n = 205.3489

n = 205

Consider a bank policy to maintain 12% of deposits as reserves. The bank currently has $10 million in deposits and holds $400,000 in excess reserves. What is the required reserve on a new deposit of $50,000?

Answers

Answer: $4,000

Explanation:

The required reserve and the excess reserve held by the bank are different because the required reserve is imposed by the central bank and the excess reserve is done by the bank.

The 12% reserves the bank is holding are both the excess and required reserves.

This means that the excess reserve is;

= 400,000/10,000,000

= 4%

The required reserve ratio will therefore be;

= 12% - 4 %

= 8%

Required reserve on $50,000 will be;

= 8% * 50,000

= $4,000

Red Sun Rising just paid a dividend of $2.52 per share. The company said that it will increase the dividend by 30% and 25 over the next two years, respectively. After that, the company is expected to increase its annual dividend at 3.8%. If the required return is 11.8%, what is the stock price today?a. $45.44.b. $42.51.c. $47.08.d. $25.00.e. $48.72.

Answers

Answer: e. $48.72.

Explanation:

Dividend next year is = 2.52 * (1 + 30%) = $3.276

Dividend in two years = 3.276 * ( 1 + 25%) = $4.095

Then calculate the terminal value at year 2;

= (Dividend * (1 + growth rate))/ (Required return - growth rate)

= (4.095 * (1 + 3.8%)) / (11.8% - 3.8%)

= $53.132625‬

Add their present values up;

= (3.276/( 1 + 11.8%)) + (4.095 / (1 + 11.8%)) + (53.132625 / ( 1 + 11.8%))

= $48.72

You decide that structural changes must be made immediately at Holden Evan to deal with the problems caused by the three SBU marketing teams. What should you do?

Answers

Hello. This question is incomplete. The full question is:

An additional challenge facing Holden Evan is the rise in costs of raw materials. Each of the SBUs currently maintains its own relationships with suppliers and is paying different prices for the same goods. The CEO of Holden Evan tells you that this is a serious issue for the company. "We must find a way to rein in these costs. I expect you to improve the company's efficiency and find other ways to reduce costs right away.

You decide that structural changes must be made immediately at Holden Evan to deal with the problems caused by the three SBU marketing teams. What should you do?

Answer:

The ideal would be to merge all marketing teams into one.

Explanation:

I would unite the three marketing teams that make up each SBU and form a single marketing team that should work in unity and serve all product lines in a unique and no longer individual way as was done previously. In this way, the cost of production and raw materials will be unique and more comprehensive, allowing greater control over expenses and team work.

The thing that should be done will be to merge all the marketing teams into one.

From the information given, the challenge that the company is facing is the rise in costs of raw materials since each of the SBUs currently maintains its own relationships with suppliers and is paying different prices for the same goods.

Therefore, in order to rein in these costs, it's important to merge all the marketing teams into one. Thus will be important in enhancing the company's efficiency and reducing cost.

Read related link on:

https://brainly.com/question/19475477

Gardner Company expects sales for October of $256,000. Experience suggests that 40% of sales are for cash and 60% are on credit. The company collects 50% of its credit sales in the month of sale and 50% in the month following sale. Budgeted Accounts Receivable on September 30 is $71,000. What is the amount of Accounts Receivable on the October 31 budgeted balance sheet?a. $102,400.b. $128,000.c. $71,000.d. $76,800.e. $153,600.

Answers

Answer:

$76,800

Explanation:

Calculation for the amount of Accounted Receivables on the October 31 budgeted balance sheet

Accounted Receivables on the October 31 budgeted balance sheet=(60% *256,000* 50%)

Accounted Receivables on the October 31 budgeted balance sheet= $76,800

Therefore the Accounted Receivables on the October 31 budgeted balance sheet will be $76,800

The basic principle involved with expense recognition is: Multiple Choice All costs that are used to generate revenue are recorded in the period the revenue is recognized. All transactions are recorded at the exchange price. The business is separate from its owners. The business will continue to operate indefinitely unless there is evidence to the contrary.A company orders office supplies in June. Those supplies are received and paid for in July. The supplies are used in August. In which month should the company record supplies expense

Answers

Answer:

All costs that are used to generate revenue are recorded in the period the revenue is recognized

Supplies expense is recorded in August

Explanation:

Expenses are the cost that a company incurs in the process of generating revenue from an asset. As such when a business is paying cost like wages, rent, and office materials it is generating revenue from sales and services provided.

The expense recognition principle requires that a business records expenses when revenue associated with them are recognised.

An office that orders office supply and uses them in August will only recognise supply expense when there is use of the office supply.

So supply expense will be recognised in August

Please answer ASAP

Barney bought a small retail business a month ago. He plans to advertise it with the help of billboards and flyers to attract customers. Which type of expense would Barney’s business incur in the form of advertising?

A. Non operating expense
B. Selling expense
C. Administration expense
D. Logistic expense
E. Production expense

Answers

Answer:

The answer is B.Selling expense

It's D Logistic Expense

(Fill in the blank)

You first calculate your total income, by adding income generated from all your sources of income. From the sum, deduct your (_____) expenses for a specific period.
The surplus amount derived is your discretionary income.

Answers

Expensive is the answer

Answer:

I believe it is average and then surplus

Explanation:

Correct me if im wrong but i think these are the answeres becasue you should subtract your average expenses and the surplus is your income

What should you do during the development phase regardless of the type of event you are implementing?

a) identify your goals and objectives

b) identify the charity for the event profits

c) identify the location of the event

d) identify who will be the master of ceremonies

Answers

Answer:

d) identify who will be the master of ceremonies

Explanation:

a) identify your goals and objectives  ⇒ Initiation phase b) identify the charity for the event profits   ⇒ Initiation phase c) identify the location of the event   ⇒ Planning or design phase

One of the first activities carried out during the operations or development phase should be to establish his/her work team (including master of ceremonies).

Emily is journalizing an adjustment for a returned check written by a customer. What type of account would this be considered?

Answers

Answer:

When you are reconciling a bank account and you receive an NSF check (non-sufficient funds), you must reverse the collection of the check:

When you received the check the journal entry was:

Dr Cash xyz

    Cr Accounts receivable xyz

If the check bounces, the reversal entry is:

Dr Accounts receivable xyz

    Cr Cash xyz

First City Bank pays 7 percent simple interest on its savings account balances, whereas Second City Bank pays 7 percent interest compounded annually.If you made a $64,000 deposit in each bank, how much more money would you earn from your Second City Bank account at the end of 9 years? (Do not round intermediate calculations and round your final answer to 2 decimal places. (e.g., 32.16))

Answers

Answer:

$13,341.39

Explanation:

The computation is shown below:

For First city bank,

The Future value is

= $64,000 + $64,000 × 7%× 9

= $64,000 + $40,320

= $104,320

And  for Second city bank,

Given that

NPER = 9,

RATE = 7%,

PV = $64,000

PMT = $0

The formula is shown below:

=-FV(RATE;NPER;PMT;PV;TYPE)

After applying the above formula, the future value is $117,661.39

Now finally

More money would be

= $117,661.39- $104,320

= $13,341.39

Consider the recorded transactions below.

Credit Debit

Accounts Receivable 7,900

Service Revenue 7,900

Supplies 2,050

Accounts Payable 2,050

Cash 9,700

Accounts Receivable 9,700 .

Advertising Expense 1,100

Cash 1,100

Accounts Payable 3,200

Cash 3,200

Cash 1,200

Deferred Revenue 1,200

Required:

Post each transaction to T-accounts and compute the ending balance of each account. The beginning balance of each acc the transactions is: Cash, $2,900; Accounts Receivable, $3,700; Supplies, $350; Accounts Payable, $3,000; Deferred Reve Service Revenue and Advertising Expense each have a beginning balance of zero.

Answers

Answer:

gogle know the answer that is the secret

A $10,000 bond with 18%/year, compounded semi-annually (interest is paid every six month) is available in the market. The bond matures in 10 years. The closest PW of this bond if the purchaser can earn 12%/year, compounded quarterly is:_____.

Answers

Answer:

present value = 13313.50

Explanation:

given data

bond = $10,000

interest = 18% per year = 9% per semi annual period

bond matures = 10 years = 20 semi annual period

purchaser can earn =  12% per year compounded quarterly = 3% per quater

so efftive semi annual rate is

efftive semi annual rate = (1+3%)² - 1  

efftive semi annual rate = 6.09%

and

we get here coupon paymnet that is

coupon payment = 9% × $10,000  

coupon payment = $900

so present value is

present value = 900 (P/A,6.09%,20) + 10000 (P/F,6.09%,20)

present value = 900 × 11.3865  +  10000 × 0.3065

present value = 13313.50

A toll on a congested road is in essence a. an interstate highway subsidy. b. a corrective tax. c. a hidden tax. d. a gasoline tax.

Answers

Answer:

b. a corrective tax.

Explanation:

Taxation can be defined as the involuntary or compulsory fees levied on individuals or business entities by the government to generate revenues used for funding public institutions and activities.

There are three (3) types of taxation used by the government, these are;

1. Progressive taxation: it involves charging individuals having higher incomes a higher percentage of their total income.

For instance, Citizen X pays 30% on $70,000 and Citizen B pays 10% on $45.000.

2. Proportional taxation: it involves charging both lower and higher income earners equally in proportion to their income.

For instance, Citizen A pays 20% on $50,000 and Citizen B pays 20% on $36,000.

3. Regressive taxation: it involves charging individuals with low incomes a higher percentage of their total income and vice-versa.

For instance, Citizen A pays 15% on $50,000 and Citizen B pays 20% on $36,000.

A corrective tax can be defined as a type of taxation imposed by the government on its citizens to correct negative externalities such as facilities, services or amenities that have socially harmful effects or capable of causing damages to the people.

Hence, a toll on a congested road is in essence a corrective tax.

A toll on a congested road is considered a corrective tax as it aims to correct market inefficiencies by discouraging excessive use and reducing congestion, while also generating revenue for infrastructure maintenance and improvements. Therefore, option a is correct.

A toll on a congested road is considered a corrective tax. It is implemented to correct market inefficiencies by discouraging excessive use of the road and reducing congestion.

By charging a toll, the aim is to internalize the negative externalities associated with congestion and encourage more efficient use of the road network.

Additionally, the revenue generated from the toll can be used for infrastructure maintenance and improvements. Thus, a toll on a congested road serves as a corrective measure to address congestion issues while also generating revenue.

Learn more about corrective tax here:

https://brainly.com/question/30724843

#SPJ6

Schedule of cash payments for a service company Horizon Financial Inc. was organized on February 28. Projected selling and administrative expenses for each of the first three months of operations are as follows:
March $160,800
April 152,800
May 139,000
Depreciation, insurance, and property taxes represent $35,000 of the estimated monthly expenses. The annual insurance premium was paid on February 28, and property taxes for the year will be paid in June. 73% of the remainder of the expenses are expected to be paid in the month in which they are incurred, with the balance to be paid in the following month.
Prepare a schedule of cash payments for selling and administrative expenses for March, April, and May.

Answers

Answer:

Total Cash Payments are as follows:

For March = $91,834

For April = $119,960

For May = $107,726

Explanation:

Note: See the attached Excel file for the schedule of cash payments

The expenses paid in each month are estimated as follows:

a. March Expenses

Paid in March = (Total projected selling and administrative expenses for March - Depreciation, insurance, and property taxes for March) * Percentage of reminder paid = ($160,800 - $35,000) * 73% = $91,834

Paid in April = (Total projected selling and administrative expenses for March - Depreciation, insurance, and property taxes for March) * Percentage of balance paid = ($160,800 - $35,000) * (100% - 73%) = $33,966

b. April Expenses

Paid in April = (Total projected selling and administrative expenses for April - Depreciation, insurance, and property taxes for April) * Percentage of reminder paid = ($152,800 - $35,000) * 73% = $85,994

Paid in May = (Total projected selling and administrative expenses for April - Depreciation, insurance, and property taxes for April) * Percentage of balance paid = ($152,800 - $35,000) * (100% - 73%) = $31,806

c. May Expenses

Paid in May = (Total projected selling and administrative expenses for May - Depreciation, insurance, and property taxes for May) * Percentage of reminder paid = ($139,000 - $35,000) * 73% = $75,920

Other Questions
Alright I need another one before I fail again please I have 3 can some one please help me im confused on this im really stuck :( Based on this passage, which word best characterizesDave?Several times (Dave) fell down and was dragged in thetraces, and once the sled ran upon him so that he limpedthereafter in one of his hind legs. But he held out tillcamp was reached, when his driver made a place for himby the fire, Morning found him too weak to travel. Athamess-up time he tried to crawl to his driver,-The Call of the Wild,Jack LondonscareddeterminedindependentinconsiderateIntroDone Please answer i wanna get this over with h ^-2 / h ^4 41 divide by 278.8 hi my name chavoria 1 There are 4 trucks for every 5 cars in a parking lot. How many trucks andcars could be in the parking lot?A 64 trucks and 80 carsB72 trucks and 73 carsC.84 trucks and 100 carsD.96 trucks and 110 cars ASAP PLS HELP ILL GIVE BRAINLIEST Which equation best represents the constant rate of change shown in the pattern between thefigure number, f, and the number of squares, s? Select the correct answer.Mel created a scatter plot and drew a line of best fit, as shown Who else hate khan academy? Examine circle O, where chords AB and CD are congruent.Point P lies on AB and point Q lies on CD.Angles APO and DQO are right angles.The diagram as described in the problem.Which congruence statement is true?A.) QDQOB.) OPABC.) CDAPD.) OPOQ Graph the liney=-4x+2will make brainliest Divide. Express your answer in simplest form. 14 49 98 28 explain why tge plant roots would take in more water after the plant has turned 1000 sucrose molecules into 2000 glucoses. Use the terms osmosis, concentration gradient, hypertonic, hypotonic 2) Find the unknown sizes. What is the value of -6 1/2 x 1 1/4 1. To what extent is Goth subculture important and accessible in society? 2. To what extent do subcultures have support? 3. To what extent can goth subculture identity be represented? 4. How does Goth influence your identity? please can someone answer me 6. Running consumes 700 calories per hour. About how many calories are used in 40 minutes of running? HELP ASAP! Relations and Functions!!! Giving brainliest!!! Which triangle shows the incenter at point A?